5
$\begingroup$

Let $E$ be an arbitrary Banach space and let $T:E^{*}\rightarrow\ell^{2}$ be a linear continuous operator. Is it true that $T$ must be the $so$-limit (i.e., limit w.r.t. the strong operator topology) of a net $(S_{d})^{*}$ $\left(d\in\mathcal{D}\right)$ of adjoint operators, with $S_{d}$:$\ell^{2}$ $\rightarrow$ $E$ and $||S_{d}||\leq||T||$ $\left(d\in\mathcal{D}\right)$?

I guess not, say $E=c_{0}(I)$ and $T$ is a surjection, where $I$ has a "big" cardinality. But maybe I'm wrong.

Any help will be highly appreciated.

$\endgroup$
2
  • 1
    $\begingroup$ Incidentally, your entire first paragraph probably fits in a MathOverflow title. I don't know enough Banach theory to know what words are reasonable to leave out, but I recommend that you rewrite the title into a question. Even "Is a continuous linear operator of Banach spaces the strong-operator limit of a net of adjoint operators?" is probably reasonable, although clearly it drops / gets wrong a number of conditions. $\endgroup$ Apr 28, 2010 at 3:03
  • $\begingroup$ @ Theo Johnson-Freyd I think a complete question would be too long as a title... "Strong operator limits of adjoint operators in Banach Spaces" would be acceptable ? If not, you can give any "fancy" title you want. Incidentally, did you read mathoverflow.net/questions/7751/… ? $\endgroup$
    – Ady
    Apr 28, 2010 at 4:02

4 Answers 4

7
$\begingroup$

You have that $T^*:(\ell^2)^* \rightarrow E^{**}$, so using that $(\ell^2)^*$ is isomorphic to $\ell^2$ (just in the linear sense, as we already have a co-ordinate system), we can regard $T^*$ as a map $\ell^2\rightarrow E^{**}$.

By the Principle of Local Reflexivity (I've used a paper of Behrends in the past, which is overkill, but is freely available: http://matwbn.icm.edu.pl/ksiazki/sm/sm100/sm10022.pdf Or look in a book on Banach space theory) for each triple $i=(M,N,\epsilon)$, where $M\subseteq E^{**}$ and $N\subseteq E^*$ are finite-dimensional, and $\epsilon>0$, we can find an operator $S_i:M\rightarrow E$ such that $(1-\epsilon)\|x\| \leq \|S_i(x)\| \leq (1+\epsilon)\|x\|$ for $x\in M$, and with $\phi(S_i(x)) = x(\phi)$ for $x\in M$ and $\phi\in N$.

So, let $P_n:\ell^2 \rightarrow \ell^2$ be the projection onto the first $n$ co-ords, let $M\supseteq T^*(P_n(\ell^2))$ and let $i=(M,N,\epsilon)$, so $S=S_i T^* P_n$ makes sense, and is a map $\ell^2\rightarrow E$. Then, for $a\in\ell^2$ and $\phi\in E^*$, $$S^*(\phi)(a) = \phi(S_i T^* P_n(a)) \rightarrow T^*(a)(\phi) = T(\phi)(a),$$ as $i$ and $n$ increase.

So we have found a bounded net $(S_d)$ (we can even choose it bounded by $\|T\|$ be rescaling a little) with $S_d^* \rightarrow T$ in the weak operator topology. But now a standard trick (take convex combinations, as the closure of a convex set is the same in the weak and norm topologies) allows us to find a net which converges SOT.

I think the proof would work for any Banach space F replacing $\ell^2$, as long as we can find a bounded net of finite rank operators $(F_\alpha)$ with $F_\alpha\rightarrow 1$ SOT. That is, F should have the bounded approximation property.

$\endgroup$
4
$\begingroup$

As Matt said, the result is true with $\ell_2$ replaced by any dual Banach space $X^*$ that has the metric approximation property (MAP). It is necessary that $X$ has the MAP. For simplicity, assume that $X$ is separable. Then there is a James-Lindenstrauss space $Y$ so that $Y^{**} = Y\oplus X^*$ with the projection $P$ onto $X^*$ having norm one, $Y^*$ has the MAP (even a monotone basis), and the embedding of $X^*$ into $Y^{**}$ is isometric and a weak$^*$ to weak$^*$ homeomorphism. Since $Y^*$ has the MAP, norm one operators into $Y^*$ are strongly approximable by norm one finite rank operators. Thus if $P$, considered as an operator from $Y^{**}$ to $X^*$, were strongly approximable by norm one dual operators, then $P$ would be weak$^*$ approximable by norm one finite rank dual operators and you would thus get the identity on $X^*$ weak$^*$ approximable by norm one finite rank dual operators, which is to say you would have norm one finite rank operators on $X$ that converge weakly to the identity on $X$. Pass to convex combinations of these to see that $X$ must then have the MAP.

I don't know what can happen when $X$ has the MAP but $X^*$ fails the MAP.

$\endgroup$
0
$\begingroup$

See, for the beginning, arXiv:1002.3902v1, section 3. I will prepare the more or less full answers to all questions from above.

Oleg Reinov, S. Petersburg University

$\endgroup$
0
$\begingroup$

An additional reference that has been published afterwards:

Reinov, O.I., Approximation of operators in dual spaces by adjoint operators, J. Math. Sci., New York 173, No. 5, 632-642 (2011); translation from Probl. Mat. Anal. 54, 165-174 (2011). ZBL1300.46020.

$\endgroup$

Your Answer

By clicking “Post Your Answer”, you agree to our terms of service and acknowledge you have read our privacy policy.

Not the answer you're looking for? Browse other questions tagged or ask your own question.